AAFP Random 19

Lakukan tugas rumah & ujian kamu dengan baik sekarang menggunakan Quizwiz!

An 18-year-old white female presents with small, localized warts on the vulva and lower vaginal mucosa. She wants to avoid injections and surgical treatment if possible. Which one of the following is an acceptable topical agent for treating these vaginal lesions? (check one) A. Trichloroacetic acid B. Podofilox gel (Condylox) C. Imiquimod cream (Aldara) D. Interferon E. Podophyllin 25% solution in alcohol (Podocon-25, Podofin)

A. Trichloroacetic acid. Trichloroacetic acid is acceptable for use on vaginal mucosa. It is also acceptable for use when pregnancy is a possibility. Professional application is necessary. Podofilox and podophyllin in alcohol are not safe for use on mucosa. Imiquimod cream is also not approved for mucosal use. Interferon requires injection.

A male infant is delivered by cesarean section because of dystocia due to macrosomia. Apgar scores are 8 at 1 minute and 10 at 5 minutes. However, at about 1 hour of age he begins to have tachypnea without hypoxemia. A chest radiograph shows diffuse parenchymal infiltrates and fluid in the pulmonary fissures. The symptoms resolve without treatment within 24 hours. The most likely diagnosis is (check one) A. transient tachypnea of the newborn B. intracranial hemorrhage C. laryngotracheomalacia D. meconium aspiration syndrome E. hyaline membrane disease

A. transient tachypnea of the newborn. This child had transient tachypnea of the newborn, the most common cause of neonatal respiratory distress. It is a benign condition due to residual pulmonary fluid remaining in the lungs after delivery. Risk factors include cesarean delivery, macrosomia, male gender, and maternal asthma and/or diabetes mellitus. The other conditions listed cause neonatal respiratory distress, but do not resolve spontaneously. They also cause additional significant abnormal findings on physical examination and/or ancillary studies such as imaging and laboratory studies.

A 32-year-old white male teacher is seen for a paroxysmal cough of 5 days duration. He tells you that a student in his class was diagnosed with pertussis 3 weeks ago. Which one of the following would be the best treatment? (check one) A. Amoxicillin B. Azithromycin (Zithromax) C. Cephalexin (Keflex) D. Ciprofloxacin (Cipro) E. Doxycycline

B. Azithromycin (Zithromax). Macrolides are considered first-line therapy for Bordetella pertussis infection. Trimethoprim/sulfamethoxazole is considered second-line therapy.

Which one of the following is associated with the use of epidural anesthesia during labor and delivery? (check one) A. A shorter first stage of labor B. A longer second stage of labor C. An increased rate of cesarean delivery D. An increased likelihood of postpartum urinary incontinence

B. A longer second stage of labor. Studies have shown that epidural analgesia increases the length of both the first and second stage of labor. Although there is an increase in the rate of instrument-assisted delivery and fourth degree laceration, an increase in the rate of cesarean sections has not been shown. An increase in the rate of urinary incontinence also has not been shown.

Patients with obstructive sleep apnea have an increased risk for (check one) A. chronic renal failure B. hypertension C. hypokalemia D. hypothyroidism E. sepsis

B. hypertension. Obstructive sleep apnea-hypopnea syndrome is defined as the presence of at least five obstructive events per hour with associated daytime sleepiness. It is present in 2%-4% of the population. The prevalence in men is almost three times that seen in premenopausal women and twice that of postmenopausal women. Other factors associated with an increased prevalence are obesity, older age, and systemic hypertension.

The definition of post-term pregnancy is a pregnancy that has reached: (check one) A. 40 weeks' gestation B. 41 weeks' gestation C. 42 weeks' gestation D. 43 weeks' gestation

C. 42 weeks' gestation. Postdate and post-term pregnancy are terms that are used interchangeably. The postdate pregnancy is defined as a pregnancy that has reached 42 weeks of amenorrhea. This is important because perinatal mortality doubles at 42 weeks gestational age. The diagnosis of postdate pregnancy depends heavily on accurate dating methods.

A 37-year-old gravida 3 para 2 at 33 weeks' gestation reports the onset of brisk vaginal bleeding. On examination the uterus is nontender and 32 cm above the symphysis. Pelvic examination reveals the presence of a large amount of bright red vaginal blood. This presentation is most consistent with: (check one) A. Threatened abortion B. Hemorrhagic cystitis C. Placenta previa D. Chorioamnionitis E. Abruptio placentae

C. Placenta previa. The classic clinical presentation of placenta previa is painless, bright red vaginal bleeding. This diagnosis must be considered in all patients beyond 24 weeks' gestation who present with bleeding. Threatened abortion is unlikely at this stage of pregnancy and hemorrhagic cystitis is not accompanied by brisk bleeding. Abruption of the placenta is the most common cause of intrapartum fetal death but is associated not only with brisk vaginal bleeding, but also with uterine tenderness that may be marked. Clinical signs of chorioamnionitis include purulent vaginal discharge, fever, tachycardia, and uterine tenderness.

A 38-year-old widow consults you 2 years after her husband's accidental death. She is planning to remarry and asks about the possibility of resuming the low-dose oral contraceptives she took before she was widowed. Which one of the following may contraindicate resumption of oral contraceptives? (check one) A. Her 42-year-old sister has breast cancer B. Her blood pressure is 135/88 mm Hg C. She smokes a pack of cigarettes each day D. She has a history of migraines resistant to triptans E. Her LDL/HDL ratio is 2.8

C. She smokes a pack of cigarettes each day. Oral contraceptives increase the risk of venous thromboembolic phenomena. The combination of oral contraceptives and smoking substantially increases the risk of cardiovascular disease. Caution should be exercised in prescribing oral contraceptives for women older than 35 years of age who smoke. In general, oral contraceptive use is considered absolutely contraindicated in women older than 35 who are heavy smokers. Women who smoke fewer than 15 cigarettes a day and patients with mildly elevated blood pressure and elevated lipid levels are not at increased risk for cardiovascular disease when oral contraceptives are used.

After fitting a 30-year-old gravida 2 para 2 for a diaphragm, you advise her not to leave the diaphragm in place for longer than 24 hours because of the risk of which one of the following? (check one) A. Loss of contraceptive effectiveness B. Chlamydia infection C. Toxic shock syndrome D. Human papillomavirus (HPV) infection E. Adhesions

C. Toxic shock syndrome. Much like with tampons, leaving diaphragms in place for more than 24 hours is associated with toxic shock syndrome.

A 22-year-old competitive cross-country skier presents with a complaint of not being able to perform as well as she expects. She has been training hard, but says she seems to get short of breath more quickly than she should. She also coughs frequently while exercising. A review of systems is otherwise negative. Her family history is negative for cardiac or pulmonary diseases. Her physical examination is completely normal, and pulmonary function tests obtained before and after bronchodilator use are normal. After you discuss your findings with the patient, she acknowledges that her expectations may be too high, but can think of no other cause for her problem. Which one of the following would be the next reasonable step? (check one) A. An echocardiogram to look for cardiomyopathy or valvular dysfunction B. Counseling regarding competition stress and athlete burnout syndrome C. A sports medicine consultation to evaluate her training regimen D. A trial of inhaled albuterol (Proventil) for exercise-induced bronchospasm

D. A trial of inhaled albuterol (Proventil) for exercise-induced bronchospasm. Exercise-induced bronchoconstriction (EIB) is a very common and underdiagnosed condition in athletes. It is defined as a 10% lowering of FEV1 when challenged with exercise. The exercise required to cause bronchoconstriction is 5-8 minutes at 80% of maximal oxygen consumption. EIB is much more common in high-ventilation sports, such as track and cross-country skiing. It is also more common in winter sports, because of the inspiration of cold, dry air. In some studies the incidence among cross-country skiers is as high as 50%, and 40% of those who have positive tests for bronchospasm are unaware of the problem. A physical examination, as well as pulmonary function tests at rest and before and after bronchodilators, will be normal unless there is underlying asthma. Among athletes with EIB, 10% will not have asthma. Bronchoprovocative testing can be ordered, but if it is not available a trial with an albuterol inhaler is reasonable. Cardiomyopathy or valvular dysfunction not found during the physical examination is possible, but much less likely. Psychological stresses are also a possible etiology, but should not receive undue attention, especially when simple questioning is not productive and more likely diagnoses have not been ruled out. Poor training methods are also possible, but in a competitive athlete this is not the most likely cause.

A 27-year-old male presents with what he thinks is a sinus infection. He has a 2-day history of right maxillary pain associated with nasal congestion and clear rhinorrhea. The only significant findings on examination are a low-grade fever and subjective tenderness with palpation over the right maxillary sinus. Which one of the following treatments is most supported by current evidence? (check one) A. Antihistamines B. Oral decongestants C. Topical vasoconstrictor sprays D. Oral analgesics E. Nasal lavage

D. Oral analgesics. Although oral antibiotics are overwhelmingly prescribed as initial treatment in acute sinusitis, it has been shown that the majority of acute illnesses are viral in origin and that 98% of cases will resolve spontaneously. Analgesics are considered the mainstay of therapy for acute sinusitis, according to evidence-based recommendations (SOR A). Other treatments should be considered if symptoms are prolonged (>7 days) or severe (two or more localizing symptoms or signs of serious bacterial complications). There is little evidence of effectiveness for antihistamines, oral decongestants, or vasoconstrictor sprays. There is also little evidence of effectiveness for nasal lavage in acute sinusitis, although it has an emerging role in chronic sinusitis.

A 60-year-old female has been on conjugated equine estrogens/medroxyprogesterone (Prempro) since she went through menopause at age 52. She still has her uterus and ovaries. She is having no side effects that she is aware of and is experiencing no vaginal bleeding. She is worried about the health effects of her hormone replacement therapy and asks your advice about risks versus benefits. Which one of the following would be accurate advice regarding these risks and benefits? (check one) A. The incidence of stroke is decreased B. The incidence of myocardial infarction is decreased C. The incidence of pulmonary embolus is decreased D. The incidence of breast cancer is increased E. The incidence of colorectal cancer is increased

D. The incidence of breast cancer is increased. The Women's Health Initiative Randomized Controlled Trial concluded that the health risks of hormone replacement therapy with combined estrogen plus progestin exceeded the benefits. Absolute risk reductions per 10,000 person-years attributable to estrogen plus progestin were 6 fewer colorectal cancers and 5 fewer hip fractures. However, absolute excess risks per 10,000 person-years included 7 more coronary heart disease events, 8 more strokes, 8 more pulmonary emboli, and 8 more invasive breast cancers.

The probability of pregnancy after unprotected intercourse is the highest at which one of the following times? (check one) A. 3 days before ovulation B. 1 day before ovulation C. The day of ovulation D. 1 day after ovulation E. 3 days after ovulation

B. 1 day before ovulation. There is a 30% probability of pregnancy resulting from unprotected intercourse 1 or 2 days before ovulation, 15% 3 days before, 12% the day of ovulation, and essentially 0% 1-2 days after ovulation. Knowing the time of ovulation therefore has implications not only for "natural" family planning, but also for decisions regarding postcoital contraception.

Which one of the following is the first-line antibiotic treatment for uncomplicated acute otitis media? (check one) A. Ceftriaxone (Rocephin) B. Amoxicillin C. Azithromycin (Zithromax) D. Cefuroxime (Ceftin) E. Trimethoprim/sulfamethoxazole (Bactrim, Septra)

B. Amoxicillin. Amoxicillin remains the recommended first-line treatment for uncomplicated acute otitis media. Various other antimicrobial agents have not proved to be more efficacious, and are associated with more frequent side effects.

A 17-year-old white female presents with new-onset left-sided lower abdominal pain. Color flow Doppler ultrasonography, in addition to pelvic ultrasonography, would be most useful for evaluating: (check one) A. Adnexal torsion B. Pelvic abscess C. Pelvic inflammatory disease D. Ruptured ovarian cyst

A. Adnexal torsion. Color Doppler flow studies are useful for evaluating blood flow to the ovary in possible cases of adnexal or ovarian torsion. Adnexal torsion is a surgical emergency. Pelvic ultrasonography, preferably with a vaginal probe, can be beneficial in the workup of ruptured ovarian cyst, pelvic abscess, and pelvic inflammatory disease without abscess. The Doppler flow study is not required with these condition.

The intranasal live, attenuated influenza vaccine would be appropriate for which one of the following? (check one) A. A 5-year-old female who is otherwise healthy B. A 12-year-old male who has a history of severe persistent asthma C. A 21-year-old female who has a history of Guillain-Barré syndrome D. A 24-year-old female who is 24 weeks pregnant E. A 55-year-old healthy male who requests influenza vaccine

A. A 5-year-old female who is otherwise healthy. The live, attenuated influenza vaccine is an option for vaccinating healthy, nonpregnant individuals age 5-49 years. The vaccine is administered intranasally. It is not indicated in patients with underlying medical conditions, such as chronic pulmonary or cardiovascular disease, or in patients with a history of Guillain-Barré syndrome, pregnant patients, or children and adolescents who receive long-term aspirin or salicylate therapy. Patients with a history of hypersensitivity to eggs should not receive this vaccine.

Which one of the following unimmunized patients should receive two doses of influenza vaccine? (check one) A. A 5-year-old with asthma B. A 10-year-old with cystic fibrosis C. A 15-year-old with sickle cell anemia D. A 30-year-old with HIV infection E. A 65-year-old with bullous emphysema

A. A 5-year-old with asthma. Two doses of influenza vaccine are recommended for children under the age of 9 years unless they have been vaccinated previously. Children 3-8 years of age should receive one or two 0.5-mL doses of split-virus vaccine intramuscularly.

A healthy 40-year-old female presents for her annual gynecologic examination. She tells you that she also needs a tuberculin screening test for her anticipated volunteer work at the local hospital. She has had no significant illness or exposures and has been your patient for nearly 20 years. You administer a PPD test which shows 10 mm of induration on the second day. The most appropriate next step in her evaluation would be: (check one) A. A repeat PPD in 2 weeks B. A chest radiograph (two views) C. Screening liver function tests D. Isoniazid (INH) for 9 months for treatment of her latent tuberculosis infection

A. A repeat PPD in 2 weeks. In 2000, the American Thoracic Society and the Centers for Disease Control and Prevention (CDC) advocated a shift in focus from screening the general population to testing only patients at increased risk for developing tuberculosis. In some persons PPD reactivity wanes with time but can be recalled by a second skin test administered 1 week or more after the first (i.e., two-step testing). For persons undergoing PPD skin testing, such as health-care workers, initial two-step testing may preclude misclassification of persons with boosted reactions as PPD converters. In those at low risk, such as this patient, a tuberculin skin test is now considered positive only if induration is at least 15 mm. Thus, this hospital volunteer would pose little risk to the hospital population since her 10-mm reaction falls within the guidelines of a negative test. She does not require diagnostic evaluation at this time, and isoniazid therapy is not indicated.

Which one of the following is consistent with terminology used in the 2001 Bethesda System for reporting cervical cytology? (check one) A. Atypical squamous cells—cannot exclude HSIL (ASC-H) B. Atypical squamous cells of unknown significance—favor neoplastic (ASCUS—favor neoplastic) C. Atypical squamous cells of unknown significance—favor reactive (ASCUS—favor reactive) D. Atypical glandular cells of unknown significance (AGUS)

A. Atypical squamous cells—cannot exclude HSIL (ASC-H). In the 2001 Bethesda System, atypical squamous cells of unknown significance (ASCUS) was replaced by atypical squamous cells (ASC). ASC is divided into atypical squamous cells-cannot exclude HSIL (ASC-H) and atypical squamous cells of unknown significance (ASC-US). ASCUS-favor reactive has been downgraded to negative in the 2001 system. Atypical glandular cells of unknown significance (AGUS) has been replaced by atypical glandular cells (AGC).

Which one of the following is appropriate treatment for asymptomatic chlamydial infection during the second trimester of pregnancy? (check one) A. Azithromycin (Zithromax) B. Doxycycline C. Metronidazole (Flagyl) D. Levofloxacin (Levaquin)

A. Azithromycin (Zithromax). Several clinical trials suggest that 7-day regimens of erythromycin or amoxicillin, and single-dose regimens of azithromycin, are effective for treating chlamydial infections during pregnancy. Doxycycline and levofloxacin are contraindicated during pregnancy due to potential ill effects on the fetus, and metronidazole is not effective for the treatment of chlamydial infections.

A 27-year-old white female sees you for the first time for a routine evaluation. A Papanicolaou test reveals atypical glandular cells of undetermined significance (AGUS). Of the following, which one is most commonly found in this situation? (check one) A. Cervical intraepithelial neoplasia B. Endometrial hyperplasia C. An endocervical polyp D. Endometrial cancer E. Ectopic decidua

A. Cervical intraepithelial neoplasia. Clinical practice guidelines recommend that all patients with atypical glandular cells of undetermined significance (AGUS) be evaluated by colposcopy and endocervical curettage; endometrial sampling is recommended in women 35 years of age or older, and in those with AGUS favoring neoplasia or suggesting an endometrial source. Cervical intraepithelial neoplasia is the most common histologic diagnosis found in patients evaluated for AGUS.

A previously healthy 20-month-old female is brought to the urgent-care clinic during the evening with a barking cough. On examination her rectal temperature is 37.9°C (100.2°F), respiratory rate 18/min, heart rate 120 beats/min, and O2 saturation 94%. She has stridor, with mild substernal retractions only when her temperature was taken. Which one of the following would be most appropriate at this point? (check one) A. Dexamethasone, 0.6 mg/kg orally or intramuscularly as a single dose B. Guaifenesin/pseudoephedrine elixir orally until symptoms improve C. Azithromycin (Zithromax) orally for 5 days D. Observation in the clinic, and if there is improvement, a 5-day course of dexamethasone

A. Dexamethasone, 0.6 mg/kg orally or intramuscularly as a single dose. Croup is a syndrome most often caused by viruses, but can occasionally be of bacterial origin as in laryngotracheitis, laryngotracheobronchitis (LTB), laryngotracheobroncheopneumonia (LTBP), or laryngeal diphtheria. Mild croup is manifested by an occasional barking cough with no stridor at rest, and mild or absent intercostal retractions. Moderate croup presents with a more frequent barking cough, stridor with suprasternal and sternal retractions at rest, but no agitation. Severe croup includes more prominent inspiratory and expiratory stridor with agitation and distress. There is good evidence that corticosteroids produce significant improvement. The regimens studied most frequently have consisted of single-dose dexamethasone (0.6 mg/kg orally or intramuscularly), with some studies including up to four more doses over a 2-day period. Longer courses of corticosteroids have not proven to be more effective and may be harmful, leading to secondary infections. Racemic epinephrine by nebulization is indicated in severe croup. Antitussives and decongestants have not been studied and are not recommended. Antibiotics are indicated in LTB and LTBP, which can be diagnosed on the basis of crackles and wheezing on examination, or by an abnormal chest radiograph. Laryngotracheitis can sometimes be associated with a bacterial infection, but should be suspected only after a patient does not improve with corticosteroids and epinephrine.

A 20-year-old single white female who is a patient of yours was raped in her apartment at 7:00 a.m. today. She is brought to your office at 9:00 a.m. for assessment and treatment. Despite having occasional intercourse with her boyfriend, she has never used any type of contraceptive. They last had intercourse approximately 1 week ago, and the boyfriend has been out of town on business since then. The patient has a history of irregular periods, and her last normal period was approximately 2 and a half weeks ago. You note live sperm on a wet mount. In addition to many other issues that must be addressed at this visit, the patient asks about emergency contraception. Which one of the following would be accurate advice to the patient regarding this topic? (check one) A. Emergency contraception does not interfere with an established, post-implantation pregnancy B. The estrogen/progestin combination regimen appears to be more effective than the levonorgestrel-only regimen C. To be most effective, each dose of the 2-dose regimen should be administered at least 72 hours apart D. Fetal malformations have been reported as a result of the unsuccessful use of the high-dose emergency contraceptive regimen

A. Emergency contraception does not interfere with an established, post-implantation pregnancy. An FDA Advisory Committee has recommended over-the-counter marketing of Plan B, an emergency contraceptive package that contains two 0.75-mg tablets of levonorgestrel to be taken 12 hours apart. Plan B is one of the two FDA-approved products for this indication. The Preven emergency contraceptive kit includes four tablets, each containing 0.25 mg of levonorgestrel and 50 Μg of ethinyl estradiol; these are taken two at a time 12 hours apart. In a randomized, controlled trial comparing the single versus combined estrogen/progestin, the single-drug regimen was shown to be more effective. Pregnancy occurred in 11 of 976 women (1.1%) given levonorgestrel alone, and in 31 of 979 (3.2%) given ethinyl estradiol plus levonorgestrel. The proportion of pregnancies prevented, compared to the expected number without treatment, was 85% with levonorgestrel and 57% with the combination. In both regimens, the interval between individual doses is 12 hours. In this case, emergency contraception may be appropriate in the face of a possible pregnancy from previous consensual intercourse. Emergency contraception has not been found to interfere with an established post-implantation pregnancy. Furthermore, no fetal malformations have been reported as a result of the unsuccessful use of high-dose oral contraceptives for emergency contraception.

A 25-year-old primigravida presents with sharp, stabbing, left-sided pelvic pain that started yesterday, 45 days after her last menstrual period. Her past history is not remarkable, and a physical examination is normal except for moderate tenderness in the left adnexa on pelvic examination. A urinalysis is normal, as is a CBC. Her beta-hCG level is 1500 mIU/mL. Assuming no adnexal mass is seen, which one of the following transvaginal pelvic ultrasonography findings would be consistent with the highest likelihood of an ectopic pregnancy? (check one) A. Empty uterus: empty endometrial cavity with or without a thickened endometrium B. Abnormal gestational sac: anechoic intrauterine fluid collection either >10 mm in mean sac diameter or with a grossly irregular border C. Nonspecific fluid: anechoic intrauterine fluid collection <10 mm in mean sac diameter without an echogenic border D. Echogenic material: echogenic material within the endometrial cavity without a defined sac, or multiple discrete anechoic collections of various sizes divided by echogenic septations

A. Empty uterus: empty endometrial cavity with or without a thickened endometrium. At this time in the patient's pregnancy, a gestational sac should be visible on ultrasonography. An empty uterus presents the highest risk (14%) for ectopic pregnancy, while nonspecific fluid and echogenic material are associated with a 5% and 4% risk, respectively. An abnormal or normal sac is associated with no risk, with the rare exception of multiple pregnancies with one being heterotopic.

A 72-year-old male slipped on a rug in his kitchen and struck his right side against a counter. He presents several days after the fall with a complaint of ongoing pain in his flank. He has a history of chronic atrial fibrillation, which is treated with warfarin (Coumadin). His vital signs are normal. A physical examination reveals tenderness to palpation along the posterior-lateral chest wall and decreased breath sounds in the right base. Radiographs reveal two fractured ribs on the right side and a moderately large pleural effusion in the right hemithorax. Laboratory test results include a hemoglobin of 10.5 mg/dL (baseline 11.0-12.0 mg/dL) and a prothrombin time of 33.5 seconds with an INR of 3.5. Which one of the following would be the most appropriate management at this time? (check one) A. Evacuation of the pleural space B. Prophylactic antibiotics C. Open fixation of the ribs with control of bleeding D. Symptomatic treatment and close follow-up E. Use of a rib binder for 2-3 weeks

A. Evacuation of the pleural space. This patient has been clinically stable despite losing what appears to be a fair amount of blood into his pleural space after fracturing two ribs, a condition referred to as hemothorax. The treatment of choice in this condition is to remove the bloody fluid and re-expand the associated lung. This therapy is felt to decrease any ongoing blood loss by having the lung pleura put a direct barrier over the site that is bleeding. It also prevents the development of empyema or fibrosis, which could occur if the blood were to remain.

The most common manifestation of uterine rupture during labor is: (check one) A. Fetal distress B. Sudden, tearing uterine pain C. Vaginal hemorrhage D. Cessation of uterine contractions E. Regression of the fetus

A. Fetal distress. Fetal distress with prolonged, variable, or late decelerations and bradycardia is the most common, and often only, sign of uterine rupture. The other signs listed are unreliable and often absent.

A 60-year-old male with a right-sided pleural effusion undergoes thoracentesis. Analysis of the pleural fluid reveals a protein level of 2.0 g/dL and an LDH level of 70 U/L. His serum protein level is 7.0 g/dL (N 6.0-8.3) and his serum LDH level is 200 U/L (N 100-105). Based on these findings, which one of the following is the most likely diagnosis? (check one) A. Heart failure B. Pulmonary embolism C. Tuberculous pleurisy D. Malignancy E. Bacterial pneumonia

A. Heart failure. Pleural effusions may be exudates or transudates. The distinction is important for an accurate diagnosis and to help determine what further evaluations may be necessary. Lights criteria use ratios of fluid/serum values for protein and LDH. Pleural fluid/serum ratios greater than 0.6 for LDH and 0.5 for protein are indicative of exudates. In the scenario presented, both ratios are approximately 0.3; therefore, the fluid is a transudate. The list of causes for transudates is much shorter than for exudates. The vast majority of transudates are due to heart failure, with cirrhosis being the next most common cause. Once there is reasonable certainty that the fluid is a transudate, additional studies usually are not necessary. The other conditions listed result in exudative pleural effusions.

When a woman less than 50 years of age develops vulvar cancer, which one of the following associated conditions is most frequently present? (check one) A. Human papillomavirus B. Lichen sclerosus C. Diabetes mellitus D. Syphilis E. Lymphogranuloma venereum

A. Human papillomavirus. There has been an increase in vulvar cancer in women 35-65 years of age over the last decade. This increase is associated with human papillomavirus infection, particularly involving subtypes 16 and 18. Lichen sclerosus is associated with vulvar cancer in older women. Hypertension, diabetes mellitus, and obesity may coexist, but are not felt to be independent risk factors. Syphilis and other granulomatous diseases have been associated with vulvar cancer in the past; they are not currently considered to be significant risk factors, but are considered markers for sexual behavior associated with increased risk.

A 22-year-old gravida 2 para 1 presents to your office with a 1-day history of vaginal bleeding and abdominal pain. Her last menstrual period was 10 weeks ago, and she had a positive home pregnancy test 6 weeks ago. She denies any passage of clots. On pelvic examination, you note blood in the vaginal vault. The internal cervical os is open. Which one of the following best describes the patient's current condition? (check one) A. Inevitable abortion B. Completed abortion C. Threatened abortion D. Incomplete abortion E. Missed abortion

A. Inevitable abortion. Inevitable abortion is defined by bleeding, an open os, and no passage of products of conception (POCs). Bleeding also occurs with completed abortion, but the os is closed and there is complete passage of POCs. Threatened abortion also is characterized by bleeding and a closed os, but there is no passage of POCs. With incomplete abortion there is bleeding and an open os, but POCs are visualized in the os or vaginal vault. There are no symptoms with missed abortion, but there is no embryo or fetus on ultrasonography.

Which one of the following is true regarding the use of a diaphragm for contraception? (check one) A. It must be refitted if the patient gains more than 15 lb B. Use of nonoxynol-9 will prevent HIV C. Diaphragms are made only of latex D. Diaphragms are recommended for women with a history of toxic shock syndrome E. The diaphragm should be removed immediately after intercourse

A. It must be refitted if the patient gains more than 15 lb. The diaphragm is an effective method of contraception if used correctly. A weight change of more than 15 lb, pregnancy, or pelvic surgery may necessitate refitting. If used with nonoxynol-9, a diaphragm may actually increase the risk of HIV transmission. Diaphragms are made of latex, but a wide seal rim model made of silicone is available for those who are latex sensitive. Diaphragm use is contraindicated in women with a history of toxic shock syndrome. The diaphragm should remain in place for 6-24 hours after intercourse.

A 50-year-old male who is a heavy smoker asks you about vitamin supplementation to prevent cancer and cardiovascular disease. The patient is unwilling to stop smoking. According to the 2003 recommendations of the U.S. Preventive Services Task Force, which one of the following is true regarding vitamin supplementation in adults who are middle-age or older? (check one) A. Large supplemental doses of ~17beta-carotene may increase the risk of lung cancer in heavy smokers B. Beta-carotene supplementation decreases the risk of cardiovascular disease and cancer in nonsmokers C. Supplementation with vitamins A, C, and E plus folic acid decreases the risk of cardiovascular disease D. Supplementation with antioxidant combination vitamins plus folic acid decreases the risk of cancer

A. Large supplemental doses of ~17beta-carotene may increase the risk of lung cancer in heavy smokers. The U.S. Preventive Services Task Force found that beta-carotene supplementation provides no benefit in the prevention of cancer in middle-aged and older adults. In two trials limited to heavy smokers, supplementation with beta-carotene was associated with a higher incidence of lung cancer and all-cause mortality. In general, little evidence was found to determine whether supplementation of any of the mentioned vitamins reduces the risk of cardiovascular disease or cancer.

The most frequently reported symptom of vulvar cancer is which one of the following? (check one) A. Longstanding pruritus B. Bleeding C. Pain D. Discharge E. Dysuria

A. Longstanding pruritus. The most common symptom of vulvar cancer is longstanding pruritis. The other symptoms mentioned occur less frequently.

As a single measurement, which one of the following provides the most accurate estimate of gestational age by ultrasound determination during the second trimester? (check one) A. Transabdominal diameter B. Biparietal diameter C. Femur length D. Crown-rump length

B. Biparietal diameter. All of the options listed can be assessed by ultrasonography. Crown-rump length is a very accurate parameter in the first trimester, but the biparietal diameter is the most accurate parameter during the second trimester. Both have a 95% confidence level of being within 5-10 days of the actual gestational age when used at the proper time.

A 35-year-old female is planning a second pregnancy. Her last pregnancy was complicated by placental abruption caused by a large fibroid tumor of the uterus, which is still present. Which one of the following would be the most appropriate treatment for the fibroid tumor? (check one) A. Myomectomy B. Myolysis with endometrial ablation C. Uterine artery embolization D. Observation

A. Myomectomy. There are numerous options for the treatment of uterine fibroids. When pregnancy is desired, myomectomy offers the best chance for a successful pregnancy when prior pregnancies have been marked by fibroid-related complications. Endometrial ablation eliminates fertility, and there is a lack of long-term data on fertility after uterine artery embolization. Observation without treatment would not remove the risk for recurrent complications during subsequent pregnancies.

In adults, which one of the following is the most likely cause of chronic, unilateral nasal obstruction? (check one) A. Nasal septal deviation B. Foreign body impaction C. Allergic rhinitis D. Adenoidal hypertrophy

A. Nasal septal deviation. The most common cause of nasal obstruction in all age groups is the common cold, which is classified as mucosal disease. Anatomic abnormalities, however, are the most frequent cause of constant unilateral obstruction. Of these, septal deviation is the most common. Foreign body impaction is an important, but infrequent, cause of unilateral obstruction and purulent rhinorrhea. Mucosal disease is usually bilateral and intermittent. Adenoidal hypertrophy is the most common tumor or growth to cause nasal obstruction, followed by nasal polyps, but both are less frequent than true anatomic causes of constant obstruction.

You see a 17-year-old white female who has recently become sexually active. She requests oral contraceptives and you perform a brief evaluation, including blood pressure measurement. A pregnancy test is negative. She is resistant to further evaluation unless it is necessary. In addition to appropriate counseling, which one of the following should be done before prescribing oral contraceptives? (check one) A. No further evaluation at this visit unless indicated by history B. A pelvic examination and Papanicolaou test C. Screening for sexually transmitted diseases D. A breast examination

A. No further evaluation at this visit unless indicated by history. Policy statements from major organizations based on reviews of relevant medical literature support the practice of prescribing initial hormonal contraception after performing only a careful review of the medical history plus measurement of blood pressure. Requiring that patients undergo pelvic and breast examinations leads many young women to avoid this most reliable method of contraception, resulting in a much higher rate of unwanted pregnancy. Follow-up blood pressure measurements are important. Often, younger women will be willing to undergo Papanicolaou (Pap) tests and STD screening later, and periodic follow-up must be scheduled. Sexually active adolescents should have annual screening for cervical cancer and sexually transmitted diseases, but these are not necessary before prescribing oral contraceptives. The longest period of time a prescription should be given without a Pap test is 1 year, but this restriction is under study. Obviously, any history indicative of high risk would modify this approach.

A 47-year-old female presents to your office complaining of hot flashes and cold sweats of several months' duration. She is premenopausal. Which one of the following is accurate advice for this patient regarding vasomotor symptoms? (check one) A. They usually peak around the time of menopause, then decline after menopause B. Without treatment, they usually get worse each year after menopause C. They are always caused by estrogen deficiency D. Estrogen alone is recommended for therapy

A. They usually peak around the time of menopause, then decline after menopause. Vasomotor symptoms slowly increase until perimenopause, at which time they peak. The symptoms then tend to diminish after menopause. Numerous other pathologic and functional vasomotor etiologies may mimic hot flashes. Estrogen is effective in treating hot flashes but generally should not be given alone, as it increases the risk for endometrial cancer.

Which one of the following is true regarding PPD testing for tuberculosis? (check one) A. Patients who have converted within the past year should be treated, regardless of age B. In patients who previously received a BCG vaccination, the threshold for a positive test is 25 mm of induration C. Patients who test positive only on the second step of a two-step PPD test, given 2 weeks after the first test, are at high risk for development of active disease D. PPD testing is contraindicated in patients who are HIV positive

A. Patients who have converted within the past year should be treated, regardless of age. Because the risk of developing active disease is highest in patients within 2 years after conversion, recent converters should generally be treated regardless of age. BCG vaccination has a limited effect on PPD reactivity; tests should not be interpreted any differently in patients who have previously received BCG. The use of a two-step approach (i.e., retesting 1-4 weeks later in patients who initially test negative) is designed to decrease the false-negative rate of PPD testing. The significance of a positive result on either phase of the test is the same. Patients who are HIV positive are at higher risk for false-negative PPDs and active disease, but PPD testing is not contraindicated.

Which one of the following is a risk factor for endometrial cancer? (check one) A. Polycystic ovary syndrome B. Multiparity C. Late menarche D. Use of an IUD E. Use of oral contraceptives

A. Polycystic ovary syndrome. Patients with persistent hyperestrogenic states are at heightened risk for the development of endometrial cancer. The chronic anovulation and consequent hyperstimulation of the endometrium seen with polycystic ovary syndrome predispose women to endometrial hyperplasia and carcinoma. Conversely, multiparity and late menarche are protective of the endometrium. Combination oral contraceptive use seems to decrease the risk for endometrial cancer. There is no evidence that IUD use leads to endometrial cancer, and it is thought that copper-containing IUDs may in fact provide some protection against endometrial cancer.

A 7-year-old male presents with a 3-day history of sore throat, hoarseness, fever to 100 degrees (38 degrees C), and cough. Examination reveals injection of his tonsils, no exudates, and no abnormal breath sounds. Which one of the following would be most appropriate? (check one) A. Recommend symptomatic treatment B. Perform a rapid antigen test for streptococcal pharyngitis C. Treat empirically for streptococcal pharyngitis D. Perform a throat culture for streptococcal pharyngitis E. Perform an office test for mononucleosis

A. Recommend symptomatic treatment. Pharyngitis is a common complaint, and usually has a viral cause. The key factors in diagnosing streptococcal pharyngitis are a fever over 100.4 degrees F, tonsillar exudates, anterior cervical lymphadenopathy, and absence of cough. Age plays a role also, with those <15 years of age more likely to have streptococcal infection, and those 10-25 years of age more likely to have mononucleosis. The scenario described is consistent with a viral infection, with no risk factors to make streptococcal infection likely; therefore, this patient should be offered symptomatic treatment for likely viral infection. Testing for other infections is not indicated unless the patient worsens or does not improve.

A 21-year-old married Hispanic female who is using no method of contraception presents to your office for evaluation of vaginal spotting 6 weeks after her last menstrual period. Her periods have previously been regular. She has had one previous episode of pelvic inflammatory disease. A home pregnancy test is positive. Which one of the following is true in this situation? (check one) A. Serum hCG levels should double every 2-3 days if the pregnancy is viable B. Painless bleeding excludes the diagnosis of ectopic pregnancy C. Laparoscopy should be performed to exclude ectopic pregnancy D. A serum progesterone level >25 ng/mL indicates that ectopic pregnancy is likely

A. Serum hCG levels should double every 2-3 days if the pregnancy is viable. Early diagnosis of ectopic pregnancy requires a high index of suspicion. Risk factors include previous ectopic pregnancy, tubal sterilization, pelvic inflammatory disease, IUD use, and in utero exposure to diethylstilbestrol. The classic triad of missed menses, pain, and bleeding may not always be present. In early pregnancies of less than 5 weeks' gestation, serial hCG levels are helpful. Serum hCG levels double every 1.4-2 days. In a healthy pregnancy the level is expected to increase by at least 66% in 48 hours. Combining serial hCG levels with transvaginal ultrasonography is the best combination for evaluation of first-trimester problems. Serum hCG levels correlate well with sonographic landmarks. At 5 weeks' gestation in a normal pregnancy, serum hCG is >1000 mIU/mL and a gestational sac can be visualized in the uterus. Serum hCG is >2500 mIU/mL at 6 weeks and a yolk sac can be seen within the gestational sac. An hCG level of 5000 mIU/mL is compatible with visualization of a fetal pole. When the level is 17,000 mIU/mL, cardiac activity can be detected. Progesterone levels are also predictive of fetal outcome. A single level of 25 ng/mL or higher indicates a healthy pregnancy and excludes ectopic pregnancy with a sensitivity of 98%. If the level is <5 ng/mL, the pregnancy is nonviable. Assessment of fetal well-being is difficult if levels are in the intermediate range of 5-25 ng/mL.

You see a 9-month-old male with a 1-day history of cough and wheezing. He has previously been healthy and was born after an uncomplicated term pregnancy. He is up to date on his immunizations. On examination his temperature is 38.6°C (101.5°F) and his respiratory rate is 30/min. He has diffuse wheezing and his oxygen saturation on room air is 94%. Because it is midwinter, you obtain a swab for influenza, which is negative. A chest radiograph shows peribronchiolar edema. Appropriate management would include which one of the following? (check one) A. Supportive care only B. Inhaled corticosteroids C. Ribavirin (Rebetol) D. Palivizumab (Synagis) E. Supplemental oxygen

A. Supportive care only. This child has a respiratory syncytial virus (RSV) infection. Supportive care is the mainstay of therapy. If the child can take in fluids by mouth and tolerate room air, outpatient management with close physician contact as needed is reasonable, especially in the absence of significant underlying risk factors. Routine use of corticosteroids is not recommended (SOR B). Although up to 60% of infants hospitalized for bronchiolitis receive corticosteroid therapy, studies have not provided sufficient evidence to support their use. Inhaled corticosteroids have not been shown to be beneficial, and the safety of high doses in infants is unclear. Supplemental oxygen should be administered if functional oxygen saturation (SpO2) persistently falls below 90% and can be discontinued when an adequate level returns (SOR C). Antiviral therapy for RSV bronchiolitis is controversial because of its marginal benefit, cumbersome delivery, potential risk to caregivers, and high cost (SOR B). Studies of ribavirin in patients with bronchiolitis have produced inconsistent findings. Palivizumab is a preventive measure, and is not used for treatment of the active disease. It may be considered in select infants and children with prematurity, chronic lung disease of prematurity, or congenital heart disease (SOR A). If used, it should be administered intramuscularly in five monthly doses of 15 mg/kg, usually beginning in November or December (SOR C).

A 28-year-old white female presents with painful genital ulcers. She has not had any previous episodes of similar outbreaks. She is single, but has had several heterosexual relationships. She has been with her current partner for 3 years. A culture confirms a herpes simplex virus (HSV) infection. Which one of the following is true regarding her situation? (check one) A. Suppressive therapy can reduce the risk of transmission to her partner B. In the genital area, HSV type 1 infection can be differentiated clinically from HSV type 2 infection C. This outbreak is conclusive evidence of infidelity in her partner D. An HSV vaccine is available for her partner to reduce his risk of infection

A. Suppressive therapy can reduce the risk of transmission to her partner. Suppressive therapy with acyclovir, valacyclovir, or famciclovir reduces, but does not eliminate, the risk of transmission of HSV to sexual partners. HSV type 1 and HSV type 2 infections in the genital area are clinically identical. Psychological issues, including anger, guilt, low self-esteem, anxiety, and depression are common after first receiving a diagnosis of genital HSV infection. Initial clinical outbreaks of genital HSV infections are often recurrences of previous infection. Either of the partners may have had an asymptomatic infection acquired in a previous relationship. An experimental HSV type 2 vaccine has been developed, but it is ineffective in men.

A 30-year-old African-American female presents with a vaginal discharge. On examination the discharge is homogeneous with a pH of 5.5, a positive whiff test, and many clue cells. Which one of the following findings in this patient is most sensitive for the diagnosis of bacterial vaginosis? (check one) A. The pH of the discharge B. The presence of clue cells C. The character of the discharge D. The whiff test

A. The pH of the discharge. Patients must have 3 of 4 Amsel criteria to be diagnosed with bacterial vaginosis. These include a pH >4.5 (most sensitive), clue cells >20% (most specific), a homogeneous discharge, and a positive whiff test (amine odor with addition of KOH).

A 28-year-old female presents for evaluation of a persistent thin discharge, with a "fishy" odor particularly noticeable after intercourse. She has no dyspareunia or dysuria, is in a monogamous relationship, and has used oral contraceptives for many years. Physical examination reveals no vulvar, vaginal, or cervical erythema. There is a homogenous white discharge that coats the vaginal walls. The vaginal pH is 7.0 and on microscopy you note stippled epithelial cells but no hyphae or trichomonads. Which one of the following is true regarding this patient? (check one) A. The treatment of choice may interact with alcohol B. The patient's partner needs to be treated simultaneously C. The diagnosis should be confirmed with a culture D. Oral contraceptives contribute to the risk for this condition

A. The treatment of choice may interact with alcohol. The patient has the typical symptoms and signs of bacterial vaginosis. There is no need for confirmatory testing. The treatment of choice is oral metronidazole, which may cause a disulfiram-like interaction with alcohol. Treatment of the partner has not been shown to improve the outcome.

A 34-year-old female who delivered a healthy infant 18 months ago complains of a milky discharge from both nipples. She reports that normal periods have resumed since cessation of breastfeeding 6 months ago. She takes ethinyl estradiol/norgestimate (Ortho Tri-Cyclen) for birth control. A complete review of systems is otherwise negative. The most likely cause of the discharge is (check one) A. a medication side effect B. breast cancer C. a hypothalamic tumor D. hypothyroidism

A. a medication side effect. This patient has galactorrhea, which is defined as a milk-like discharge from the breast in the absence of pregnancy in a non-breastfeeding patient who is more than 6 months post partum. It is more common in women ages 20-35 and in women who are previously parous. It also can occur in men. Medication side effect is the most common etiology. The most common pharmacologic cause of galactorrhea is oral contraceptives. Oral contraceptives that contain estrogen can both suppress prolactin inhibitory factor and stimulate the pituitary directly, both of which can cause galactorrhea. Other medications that can cause galactorrhea include metoclopramide, cimetidine, risperidone, methyldopa, codeine, morphine, verapamil, SSRIs, butyrophenones, dopamine-receptor blockers, tricyclics, phenothiazines, and thioxanthenes. Breast cancer is unlikely to present with a bilateral milky discharge. The nipple discharge associated with cancer is usually unilateral and bloody. Pituitary tumors are a pathologic cause of galactorrhea due to the hyperprolactinemia that is caused by the blockage of dopamine from the hypothalamus, or by the direct production of prolactin. However, patients often have symptoms such as headache, visual disturbances, temperature intolerance, seizures, disordered appetite, polyuria, and polydipsia. Patients with prolactinomas often have associated amenorrhea. These tumors are associated with marked levels of serum prolactin, often >200 ng/mL. Hypothalamic lesions such as craniopharyngioma, primary hypothalamic tumor, metastatic tumor, histiocytosis X, tuberculosis, sarcoidosis, and empty sella syndrome are significant but infrequent causes of galactorrhea, and generally cause symptoms similar to those of pituitary tumors, particularly headache and visual disturbances. It is rare for primary hypothyroidism to cause galactorrhea in adults. Symptoms that would be a clue to this diagnosis include fatigue, constipation, menstrual irregularity, weight changes, and cold intolerance.

A 20-year-old female long-distance runner presents with a 3-month history of amenorrhea. A pregnancy test is negative, and other blood work is normal. She has no other medical problems and takes no medications. With respect to her amenorrhea, you advise her (check one) A. to increase her caloric intake B. that this is a normal response to training C. to begin an estrogen-containing oral contraceptive D. to stop running

A. to increase her caloric intake. Amenorrhea is an indicator of inadequate calorie intake, which may be related to either reduced food consumption or increased energy use. This is not a normal response to training, and may be the first indication of a potential developing problem. Young athletes may develop a combination of conditions, including eating disorders, amenorrhea, and osteoporosis (the female athlete triad). Amenorrhea usually responds to increased calorie intake or a decrease in exercise intensity. It is not necessary for patients such as this one to stop running entirely, however.

Black cohosh is: (check one) A. A form of herbal licorice with gastrointestinal effects B. A botanical medicine used to alleviate menopausal symptoms C. A type of toxic hallucinogenic mushroom D. A variety of Cannabis sativa E. A form of dried hashish

B. A botanical medicine used to alleviate menopausal symptoms. Black cohosh is an herbal preparation widely used in the treatment of menopausal symptoms and menstrual dysfunction. Studies have demonstrated that this botanic medicine appears to be effective in alleviating menopausal symptoms. It has not been proven effective in randomized controlled trials and should not be used to prevent osteoporosis. Questions as to its stimulating effect on endometrial tissue are as yet unanswered.

A 42-year-old female presents with a 2-day history of chest pain. She describes the pain as sharp, located in the right upper chest, and worsened by deep breathing or coughing. She also complains of shortness of breath. She was previously healthy and has no recent history of travel. Her vital signs are normal. A pleural friction rub is noted on auscultation of the lungs. The remainder of the examination is normal. An EKG, cardiac enzymes, oxygen saturation, and a D-dimer level are all normal. Which one of the following would be most appropriate at this point? (check one) A. No further testing B. A chest radiograph C. An antinuclear antibody test D. Echocardiography E. Pulmonary angiography

B. A chest radiograph. This patient has pleurisy. Patients presenting with pleuritic chest pain may have life-threatening disorders, and pulmonary embolism, acute myocardial infarction, and pneumothorax should be excluded. While 5%-20% of patients with pulmonary embolism present with pleuritic chest pain, this patient has no risks for pulmonary embolism and the normal D-dimer level obviates the need for further evaluation. Moderate- to high-risk patients may need a helical CT scan or other diagnostic testing. An EKG and chest radiograph are recommended in the evaluation of acute/subacute pleuritic chest pain. The chest radiograph will exclude pneumothorax, pleural effusion, or pneumonia. An echocardiogram would not be indicated if the cardiac examination and EKG are normal. An antinuclear antibody level could be considered in recurrent pleurisy or if other symptoms or signs of lupus were present, but it would not be indicated in this patient. Most cases of acute pleurisy are viral and should be treated with NSAIDs unless the workup indicates another problem.

A 6-year-old male is brought in for evaluation by his mother, who is concerned that he may have asthma. She reports that he coughs about 3 days out of the week and has a nighttime cough approximately 1 night per week. There is a family history of eczema and allergic rhinitis. Which one of the following would be the preferred initial treatment for this patient? (check one) A. A leukotriene receptor antagonist such as montelukast (Singulair) B. A low-dose inhaled corticosteroid such as budesonide (Pulmicort Turbuhaler) C. A long-acting beta-agonist such as salmeterol (Serevent) D. A mast-cell stabilizer such as cromolyn sodium (Intal)

B. A low-dose inhaled corticosteroid such as budesonide (Pulmicort Turbuhaler). The National Asthma Education and Prevention Program (NAEPP) updated its recommendations for the treatment of asthma in 2002. Treatment is based on asthma classification. This child meets the criteria for mild persistent asthma: symptoms more than 2 times per week but less than once a day, symptoms less than 2 nights per month, peak expiratory flow (PEF) or FEV1 >80% of predicted, and a PEF variability of 20%-30%. Asthma controller medications are recommended for all patients with persistent asthma, and the preferred long-term controller treatment in mild persistent asthma is a low-dose inhaled corticosteroid. Cromolyn, leukotriene modifiers, nedocromil, and sustained-release theophylline are alternatives, but are not preferred initial agents. Quick-acting, quick-relief agents such as short-acting beta-agonists are appropriate for prompt reversal of acute airflow obstruction.

A 28-year-old primigravida is at 20 weeks' gestation by dates but her fundal height is consistent with a 26-week gestation. She has had episodes of vomiting during the pregnancy that were more severe than the physiologic vomiting typically seen in pregnancy. A sonogram performed at about 5 weeks' gestation for vaginal bleeding was normal and showed a single fetus. Which one of the following would be most appropriate at this point? (check one) A. A serum hCG level B. A repeat sonogram C. MRI of the pelvis D. Expectant management

B. A repeat sonogram. Ultrasonography is the initial test of choice for evaluating the possibility of multiple gestation. It should be done if uterine size is larger than expected, or if pregnancy-associated symptoms are excessive. It should also be done in women who received fertility treatment. An initial sonogram that shows a single pregnancy does not rule out multiple gestation. In one study, 30 of 220 twin pregnancies had an original sonogram which showed a single pregnancy. Serum hCG and MRI would not be indicated at this stage in the evaluation.

A 60-year-old female is admitted to the hospital with pneumonia 1 week after her discharge following elective colorectal surgery. Her initial stay was 5 days and she had no complications. She had no signs of infection until 2 days ago when she developed a temperature of 39.1°C (102.4°F), a cough with yellow sputum, and hypoxia. She has no abdominal pain or diarrhea. Her pulse rate is slightly elevated to 96 beats/min, and her blood pressure is unchanged from baseline. A chest radiograph confirms a left lingular infiltrate. Methicillin-resistant Staphylococcus pneumonia is rare in this institution. Of the following antibiotic regimens, which one would be the best initial treatment for this patient? (check one) A. Ampicillin/sulbactam (Unasyn) B. Ceftazidime sodium (Fortaz) and gentamicin C. Ceftriaxone (Rocephin) and azithromycin (Zithromax) D. Clarithromycin (Biaxin) E. Levofloxacin (Levaquin)

B. Ceftazidime sodium (Fortaz) and gentamicin. This patient has a significant pneumonia that requires the initiation of empiric antibiotics. It is important to remember that because this patient was recently in the hospital, the usual coverage for community-acquired pneumonia is not adequate. Health care-associated pneumonia is more likely to involve severe pathogens such as Pseudomonas aeruginosa, Klebsiella pneumoniae, and Acinetobacter species. Methicillin-resistant Staphylococcus aureus also is a consideration, depending on local prevalence. Of the antibiotic regimens listed, ceftazidime and gentamicin is the only choice that covers these organisms.

A 30-year-old white male complains of several weeks of nasal stuffiness, purulent nasal discharge, and facial pain. He does not respond to a 3-day course of trimethoprim/sulfamethoxazole (Bactrim, Septra). Follow-up treatment with 2 weeks of amoxicillin/clavulanate (Augmentin) is similarly ineffective. Of the following diagnostic options, which one is most appropriate at this time? (check one) A. Pulmonary function testing B. Coronal CT of the sinuses C. Culture and sensitivity testing of the discharge D. Erythrocyte sedimentation rate

B. Coronal CT of the sinuses. This patient has a clinical presentation consistent with acute sinusitis. Failure to respond to adequate antibiotic therapy suggests either a complication, progression to chronic sinusitis, or a different, confounding diagnosis. The diagnostic procedure of choice in this situation is coronal CT of the sinuses, due to its increased sensitivity and competitive cost when compared with standard radiographs. Cultures of the nasal discharge give unreliable results because of bacterial contamination from the resident flora of the nose. The other options listed do not contribute to the diagnosis and treatment of sinusitis.

A 38-year-old white female presents to your office with a 4-cm palpable nodule in her right breast. Fine-needle aspiration yields 4 cc of bloody fluid. Following the aspiration, the breast nodule is no longer palpable. Which one of the following would be most appropriate at this point? (check one) A. No further workup B. Cytologic examination of the fluid C. Surgical referral for core needle biopsy D. Surgical referral for excisional biopsy E. Ultrasonography of the breast

B. Cytologic examination of the fluid. When straw-colored or grey-green fluid is obtained by fine-needle aspiration of a breast nodule and the lesion completely disappears, the diagnosis is simple cyst. The fluid should not be sent for analysis because the risk for cancer is exceedingly small. If the fluid is bloody or otherwise unusual, it should be sent for cytologic examination because about 7% of bloodstained aspirates are associated with cancer.

A 19-year-old white female presents for an initial family planning evaluation. Specifically, she is interested in oral contraception. She is not presently sexually active, but has a steady boyfriend. She has no contraindications to oral contraceptive use. She has mild acne vulgaris. You discuss possible side effects and benefits of combined oral contraceptives, including improvement of her acne. Which one of the following is also associated with oral contraceptive use? (check one) A. Increased risk of ovarian cancer B. Decreased risk of ovarian cysts C. Increased risk for ectopic pregnancy D. Increased incidence of dysmenorrhea

B. Decreased risk of ovarian cysts. Women who take combination oral contraceptives have a reduced risk of both ovarian and endometrial cancer. This benefit is detectable within a year of use and appears to persist for years after discontinuation. Other benefits include a reduction in dysfunctional uterine bleeding and dysmenorrhea; a lower incidence of ovarian cysts, ectopic pregnancy and benign breast disease; and an increase in hemoglobin concentration. Many women also benefit from the convenience of menstrual regularity. All combination oral contraceptives raise sex hormone-binding globulin and decrease free testosterone concentrations, which can lead to improvement in acne.

You are evaluating a 28-year-old primigravida for an abnormal Papanicolaou (Pap) test. Which one of the following procedures would be contraindicated? (check one) A. Colposcopy B. Endocervical curettage C. Human papillomavirus (HPV) testing D. Cervical staining E. A cervical biopsy

B. Endocervical curettage. Of the choices listed, only endocervical curettage is contraindicated in pregnancy. Colposcopy, cervical biopsy, cervical staining, and HPV testing can all be safely performed during pregnancy.

Uterine rupture is a potential complication of attempted vaginal birth after cesarean (VBAC). The most reliable indication that uterine rupture may have occurred is: (check one) A. Cessation of uterine contractions during active labor B. Fetal bradycardia C. Vaginal bleeding D. Sudden lower abdominal pain E. Maternal hypotension

B. Fetal bradycardia. Uterine rupture occurs in 0.2%-1.0% of women in labor after one previous low transverse cesarean section. Obviously, this can have devastating consequences for the mother and baby, so vigilance during labor is paramount. Uterine pain, cessation of contractions, vaginal bleeding, failure of labor to progress, or fetal regression may occur, but none of these are as consistent as fetal bradycardia in cases of uterine rupture during labor for VBAC patients.

A 14-year-old African-American female presents for a routine evaluation. On examination, you note a rubbery, well-defined, nontender breast mass approximately 2 cm in diameter. The patient denies any history of breast tenderness, nipple discharge, or skin changes. The most likely diagnosis is: (check one) A. Fibrocystic breast disease B. Fibroadenoma C. Benign breast cyst D. Cystosarcoma phyllodes E. Intraductal papilloma

B. Fibroadenoma. Most breast masses in adolescent girls are benign. Fibroadenoma is the most common, accounting for approximately two-thirds of all adolescent breast masses. It is characterized by a slow growing, nontender, rubbery, well-defined mass, most commonly located in the upper, outer quadrant. Size varies, and is most commonly in the range of 2-3 cm. Fibrocystic disease is found in older adolescents and is characterized by bilateral nodularity and cyclic tenderness. Benign breast cysts are characterized by a spongy, tender mass with symptoms exacerbated by menses. Cysts are frequently multiple, and spontaneous regression occurs in 50% of patients. Cystosarcoma phyllodes is a rare tumor with malignant potential, although most are benign. It presents as a firm, rubbery mass that may enlarge rapidly. Skin necrosis is usually associated with the tumor. Intraductal papillomas are usually benign but do have malignant potential. They are commonly subareolar and are associated with nipple discharge. These tumors are rare in the adolescent population.

You are the team physician for the local high-school swim team. Over the past week, seven members of the team have developed both folliculitis and outer ear infections. You suspect bacterial contamination of the swimming pool. Which one of the following is the most likely cause? (check one) A. Streptococcus pneumoniae B. Pseudomonas aeruginosa C. Corynebacterium ulcerans D. Staphylococcus epidermidis E. Escherichia coli

B. Pseudomonas aeruginosa. Athletes, including swimmers, are susceptible to a number of skin infections. The pH of the external ear is normally acidic. Continued water exposure raises the pH, creating conditions for bacterial overgrowth, most often caused by either Pseudomonas aeruginosa or Staphylococcus aureus. Swimming pool folliculitis is most often attributed to colonization of water with P. aeruginosa.

You discover a 10-cm enlarging hematoma adjacent to the episiotomy site in a patient whose baby you delivered 6 hours ago. The best management at this time is: (check one) A. A perineal pad and cold compresses B. Removal of the sutures and clots, and reclosure C. Hypogastric artery ligation D. Needle aspiration of the hematoma

B. Removal of the sutures and clots, and reclosure. Enlarging postpartum hematomas adjacent to an episiotomy are best treated by removing the sutures and ligating the specific bleeding sites. A perineal pad and cold compresses are inadequate for an enlarging lesion, and hypogastric artery ligation and hysterectomy are indicated only with supravaginal hematomas.

A 15 and a half year-old female is brought to your office by her mother. They are concerned because she has not started her periods. She has been healthy, and has grown several inches in the last year. Her height is now 152 cm (60 in) and she weighs 44 kg (98 lb). She started to develop breast buds about a year ago and has scant pubic hair. She denies sexual activity. The mother's menarche occurred at age 15. Which one of the following is true in this case? (check one) A. The patient has delayed puberty and should have her hormone levels evaluated B. The patient will likely start her periods within a year C. Oral contraceptives will be needed to trigger menarche D. A pregnancy test should be performed E. The daughter's age of menarche is unrelated to her mother's age of menarche

B. The patient will likely start her periods within a year. The changes associated with puberty occur in an orderly sequence over a definite time frame. Any deviation from this sequence or time frame should be regarded as abnormal. In girls, pubertal development typically requires 4.5 years. Although generally the first sign of puberty is accelerated growth, breast budding is usually the first recognized pubertal change, followed by the presence of pubic hair, peak growth velocity, and menarche. Girls must have adequate nutrition and reach a critical body weight and body fat percentage before menarche occurs. There is a concordance in the age of menarche in mother-daughter pairs and between sisters. Delayed or interrupted puberty is defined as failure to develop any secondary sex characteristics by age 13, to have menarche by age 16, or to have menarche 5 or more years after the onset of pubertal development.

In patients with breast cancer, the most reliable predictor of survival is (check one) A. estrogen receptor status B. cancer stage at the time of diagnosis C. tumor grade D. histologic type E. lymphatic or blood vessel involvement

B. cancer stage at the time of diagnosis. The most reliable predictor of survival in breast cancer is the stage at the time of diagnosis. Tumor size and lymph node involvement are the main factors to take into account. Other prognostic parameters (tumor grade, histologic type, and lymphatic or blood vessel involvement) have been proposed as important variables, but most microscopic findings other than lymph node involvement correlate poorly with prognosis. Estrogen receptor (ER) status may also predict survival, with ER-positive tumors appearing to be less aggressive than ER-negative tumors.

A 70-year-old male sees you because of slowly increasing problems with COPD. He has had frequent exacerbations requiring emergency department visits. He currently uses a tiotropium (Spiriva) inhaler once a day, as well as an albuterol (Proventil) inhaler, 2 puffs 4 times a day as needed. An examination shows decreased breath sounds throughout, and an oxygen saturation of 92%. Spirometry shows he has severe COPD (stage III); his FEV1/FVC ratio is 65% of predicted and his FEV1 is 45% of predicted. The most reasonable change in treatment would be to add (check one) A. oxygen, 2 L/min while sleeping B. inhaled fluticasone (Flovent), 2 puffs twice daily C. oral low-dose prednisone daily D. oral theophylline (TheoDur, Uniphyl) twice daily

B. inhaled fluticasone (Flovent), 2 puffs twice daily. This patient is suffering from severe COPD (stage III) and has a history of frequent exacerbations. The addition of a corticosteroid inhaler for patients with severe disease has been found to significantly decrease the number of exacerbations, but has no effect on overall mortality. Side effects of oral candidiasis and easy bruising of the skin are increased. Continuous oxygen has been shown to improve overall mortality and endurance in patients with an oxygen saturation of 88% or less, but has not been shown to improve quality of life in those with mild hypoxemia or if used only at night. Oral prednisone has been shown to be effective when used to treat acute exacerbations, but when used on a chronic basis it is no more effective than corticosteroid inhalers. Chronic oral prednisone is also associated with significant side effects, and therefore is not generally recommended. Oral theophylline has not been shown to be of benefit in either preventing exacerbations or improving quality of life, and has significant side effects of gastrointestinal toxicity, seizures, and arrhythmias. It should be reserved for carefully selected patients only.

A 25-year-old Hispanic male comes to the emergency department with the sudden onset of moderate to severe right-sided chest pain and mild dyspnea. Vital signs are normal. A chest film shows a loss of markings along the right lung margins, involving about 10%-15% of the lung space. The mediastinum has not shifted. The best INITIAL treatment would be (check one) A. strict bed rest B. oxygen supplementation and close observation C. decompression of the chest by insertion of a large-bore intravenous catheter into the right second intercostal space at the midclavicular line D. immediate chest tube insertion using a water seal E. thoracotomy for wedge resection of pulmonary blebs

B. oxygen supplementation and close observation. A small spontaneous pneumothorax involving less than 15%-20% of lung volume can be managed by administering oxygen and observing the patient. The pneumothorax will usually resorb in about 10 days if no ongoing air leak is present. Oxygen lowers the pressure gradient for nitrogen and favors transfer of gas from the pleural space to the capillaries. Decompression with anterior placement of an intravenous catheter is usually reserved for tension pneumothorax. Chest tube placement is used if observation is not successful or for larger pneumothoraces. Strict bed rest is not indicated.

An asymptomatic 24-year-old white female comes to your office for a refill of oral contraceptive pills. A speculum examination is normal with the exception of a slightly friable, well-demarcated, 1.4-cm raised lesion involving a portion of the cervix. All previous Papanicolaou (Pap) tests have been normal and she has no history of abnormal bleeding or leukorrhea. Which one of the following would be most appropriate at this point? (check one) A. A Pap test, including a scraping of the erosion, with routine follow-up unless the patient becomes symptomatic B. A Pap test with follow-up in 3 months if results are normal C. A Pap test and a colposcopically-directed biopsy D. A cone biopsy E. Topical antibiotic cream

C. A Pap test and a colposcopically-directed biopsy. The finding of a red, raised, friable lesion on the cervix, or a well-demarcated cervical lesion, mandates a biopsy to exclude cervical carcinoma, and treatment for chronic cervicitis should not be started until the biopsy results are available. A Papanicolaou test by itself is insufficient if there is a grossly visible lesion, as false-negatives occur in 10%-50% of tests.

Your patient is in the second stage of labor, and you determine that the fetus is in face presentation, mentum anterior. Progress has been rapid and fetal heart tones are normal. You would now: (check one) A. Perform an immediate cesarean delivery B. Proceed with midforceps delivery C. Anticipate vaginal delivery with close fetal monitoring D. Manually convert to vertex presentation

C. Anticipate vaginal delivery with close fetal monitoring. Most infants with face presentation, mentum anterior, can be delivered vaginally, either spontaneously or with low forceps. Cesarean section is indicated for fetal distress and failure to progress. Midforceps delivery is not indicated. If fetal electrodes are attached, the chin is the preferred location.

The only nonsexual behavior that is consistently and strongly correlated with cervical dysplasia and cervical cancer is: (check one) A. Alcohol consumption B. Caffeine consumption C. Cigarette smoking D. Cocaine use E. A high-fat diet

C. Cigarette smoking. Cigarette smoking is the only nonsexual behavior consistently and strongly correlated with cervical dysplasia and cancer, independently increasing the risk two- to fourfold.

A 13-year-old white female reports a 6-month history of intermittent abdominal cramping, with each episode becoming progressively worse. Based on her history, there is no obvious relationship to eating, voiding, or defecating. She reports that she has not yet begun menstruating and is not sexually active. Her weight has been stable. She appears to be in mild emotional distress about being "the last girl in her class to have a period." She is in no physical discomfort and her vital signs are normal. Secondary sexual characteristics appear to be developing normally. She is in the 57th percentile for height and the 65th percentile for weight. A complete physical examination confirms your presumptive diagnosis. The therapeutic procedure of choice would be: (check one) A. Appendectomy B. Colonoscopy C. Hymenotomy D. Cystoscopy E. Paracentesis

C. Hymenotomy. The key to making a diagnosis of imperforate hymen, aside from the obvious finding on physical examination, lies in the systematic drawing of inferences. One can speculate that this patient's recurrent crescendo abdominal cramping represented six menstrual sheddings, with no egress from the body. Her delay in menarche, despite normal growth parameters, offers another clue that structural amenorrhea is present. Amounts of retained blood vary among patients; up to 3000 mL have been reported. A large volume can accumulate without causing any permanent damage, and subsequent fertility is usually normal. Hymenotomy will relieve the pressure, and normal menses should ensue.

Which one of the following describes the McRoberts maneuver for managing shoulder dystocia? (check one) A. Suprapubic pressure B. Delivery of the posterior arm C. Maximal flexion and abduction of the maternal hips D. Rolling the mother to an "all-fours" position E. Rotation of the fetal head

C. Maximal flexion and abduction of the maternal hips. When the just-delivered fetal head retracts firmly against the perineum, shoulder dystocia is apparent. This is an obstetric emergency that requires appropriate assistance and a calm but timely approach to ensure a safe delivery. While all of the maneuvers described are steps in managing shoulder dystocia, the McRoberts maneuver by itself (maximal flexion and abduction of the maternal hips) relieves the impaction of the anterior shoulder against the maternal symphysis in a large percentage of cases, especially when combined with suprapubic pressure.

You diagnose Trichomonas vaginitis in a 25-year-old white female, and treat her and her partner with metronidazole (Flagyl), 2 g in a single dose. She returns 1 week later and is still symptomatic, and a saline wet prep again shows Trichomonas. Which one of the following is the most appropriate treatment at this time? (check one) A. Metronidazole gel 0.75% (MetroGel) intravaginally for 5 days B. Metronidazole, 2 g orally, plus metronidazole gel 0.75% intravaginally for 5 days C. Metronidazole, 500 mg orally twice a day for 7 days D. Clindamycin cream (Cleocin) 2% intravaginally for 7 days E. Sulfadiazine (Microsulfon), 4 g orally in a single dose, plus pyrimethamine (Daraprim), 200 mg orally in a single dose

C. Metronidazole, 500 mg orally twice a day for 7 days. The preferred treatment for Trichomonas vaginitis is metronidazole, 2 g given in a single oral dose. Certain strains of Trichomonas vaginalis, however, have diminished sensitivity to metronidazole. Patients who fail initial treatment with metronidazole should be retreated with 500 mg orally twice a day for 7 days. If treatment fails again, the patient should be treated with 2 g daily for 3-5 days. Metronidazole gel and clindamycin cream are useful for treating bacterial vaginosis, but are not effective in the treatment of Trichomonas vaginitis. Sulfadiazine and pyrimethamine are used to treat toxoplasmosis.

A 30-year-old female presents with concerns about vaginal bleeding. She states that her menstrual periods have occurred at regular intervals of 28-30 days for the past 15 years, but recently bleeding has also occurred for a day or two in the middle of her cycle. This bleeding has been heavy enough to require the use of multiple pads. Which one of the following terms best describes her bleeding pattern? (check one) A. Polymenorrhea B. Mid-cycle spotting C. Metrorrhagia D. Menometrorrhagia E. Acute emergent abnormal uterine bleeding

C. Metrorrhagia. This patient has metrorrhagia, or "bleeding intermenstrual," characterized by bleeding heavy enough to require the use of multiple pads; the heavy bleeding occurs between normal menstrual bleeding. It is important to evaluate metrorrhagia because potential causes include cervical disease, problems with IUDs, endometritis, polyps, submucous myomas, endometrial hyperplasia, and cancer. Mid-cycle spotting, as the term implies, refers to light spotting and is often caused by a decline in estrogen levels. Polymenorrhea is bleeding occurring at intervals of less than 21 days. Menometrorrhagia is heavy and/or prolonged bleeding occurring at irregular, noncyclic intervals. Acute emergent abnormal uterine bleeding is characterized by significant blood loss resulting in hypovolemia.

During a routine physical examination of a 35-year-old Asian female, you note a right adnexal fullness. She has had no symptoms of pain or bloating and has been menstruating normally. Her menses occur approximately every 30 days and her next period is expected to occur in 1 week. Pelvic ultrasonography reveals a thin-walled simple cyst 5 cm in diameter. No other abnormalities are seen in the pelvic structures. Which one of the following is the best course of management for this condition? (check one) A. Reassurance only B. Checking for any increase in adnexal fullness at her next annual physical examination C. Repeat ultrasonography in 2-3 months to confirm resolution of the cyst D. Referral for ultrasound-guided aspiration of the cyst E. Referral for laparoscopic removal of the cyst

C. Repeat ultrasonography in 2-3 months to confirm resolution of the cyst. Adnexal masses in women under 45 years of age are benign in 80%-85% of cases. The specific findings of this case also strongly suggest a benign etiology, namely a thin-walled, simple cyst, a lesion that is less than 8 cm in size, and a patient of relatively young age. No aggressive means are indicated in these situations unless there are significant clinical symptoms such as pain, abdominal pressure, urinary symptoms, or gastrointestinal symptoms. Most experts currently recommend a conservative approach with repeat ultrasonography in at least 2 months, during which time the vast majority of benign cysts resolve spontaneously.

Which one of the following is the most correct recommendation regarding seat belt use for a woman at 38 weeks' gestation? (check one) A. The seat belt should be positioned over the dome of the uterus and the shoulder harness should be positioned between the breasts B. The seat belt should be positioned under the abdomen over both the anterior superior iliac spines and the pubic symphysis; the belt should be applied with some slack C. The seat belt should be positioned under the abdomen over both the anterior superior iliac spines and the pubic symphysis; the shoulder harness should be positioned between the breasts; the belt should be applied as snugly as comfort will allow D. Seat belts, should not be used in the later stages of pregnancy

C. The seat belt should be positioned under the abdomen over both the anterior superior iliac spines and the pubic symphysis; the shoulder harness should be positioned between the breasts; the belt should be applied as snugly as comfort will allow. Pregnant women can and should always wear a seat belt when driving or riding in a car. The seat belt should be positioned under the pregnant woman's abdomen over both the anterior superior iliac spines and the pubic symphysis. The shoulder harness should be positioned between the breasts.

A 31-year-old married white female complains of vaginal discharge, odor, and itching. Speculum examination reveals a homogeneous yellow discharge, vulvar and vaginal erythema, and a "strawberry" cervix. The most likely diagnosis is: (check one) A. Candidal vaginitis B. Bacterial vaginosis C. Trichomonal vaginitis D. Chlamydial infection E. Herpes simplex type 2

C. Trichomonal vaginitis. Trichomonal vaginitis usually causes a yellowish discharge which sometimes has a frothy appearance. Colpitis macularis (strawberry cervix) is often present. Monilial vaginitis classically causes a cheesy, whitish exudate with associated vaginal itching and burning. There may be vaginal and vulvar erythema and edema, but colpitis macularis is not a feature. Bacterial vaginosis is characterized by a grayish discharge with few other physical signs or symptoms, if any. Chlamydia may cause a yellowish cervical discharge and symptoms of pelvic inflammatory disease or, alternatively, may be totally asymptomatic. Herpes simplex type 2 causes ulcerations on the vulva and vaginal mucosa which are exquisitely tender, often with marked surrounding erythema and edema.

Which one of the following indicates that a patient has entered the second stage of labor? (check one) A. A small amount of bloody, mucous discharge from the cervix ("bloody show") B. Braxton Hicks contractions C. Spontaneous rupture of the chorioamnionic membranes D. Complete dilation of the uterine cervix E. Successful delivery of the placenta

D. Complete dilation of the uterine cervix. For many women, labor will be preceded by several hours, or even days, by "bloody show." So-called "false labor," or Braxton Hicks contractions, consists of weak, irregular, regional contractions that usually occur for weeks before the onset of actual labor and abate with time, analgesia, and sedation. Spontaneous chorioamnionic membrane rupture precedes the onset of labor in about 10% of pregnancies, and amniotic fluid leaks through the cervix and out the vagina. The second stage of labor is defined as the period from complete cervical dilation to complete delivery of the baby. When the cervix is completely dilated, the patient usually experiences the urge to push with contractions. The third stage of labor begins with the delivery of the baby and ends with the delivery of the placenta.

Which one of the following fetal ultrasound measurements gives the most accurate estimate of gestational age in the first trimester (up to 14 weeks)? (check one) A. Femur length B. Biparietal diameter C. Abdominal circumference D. Crown-rump length E. Scapulo-sacral length

D. Crown-rump length. Because the growth pattern of the fetus varies throughout pregnancy, the accuracy of measurements and their usefulness in determining gestational age and growth vary with each trimester. Crown-rump length is the distance from the top of the head to the bottom of the fetal spine. It is most accurate as a measure of gestational age at 7-14 weeks. After that, other measurements are more reliable. In the second trimester, biparietal diameter and femur length are used. During the third trimester, biparietal diameter, abdominal circumference, and femur length are best for estimating gestational age.

Which one of the following is recommended for the treatment of intravaginal genital warts in pregnant women? (check one) A. Imiquimod 5% cream (Aldara) B. Podofilox 0.5% solution (Condylox) C. Podophyllin 10%-25% in tincture of benzoin (Podofin) D. Cryotherapy with liquid nitrogen E. Interferon-alpha

D. Cryotherapy with liquid nitrogen. Genital warts can proliferate and fragment during pregnancy, and many specialists recommend that they be eliminated. Imiquimod, podophyllin, and podofilox are not recommended for use during pregnancy. For the treatment of vaginal warts, the Centers for Disease Control and Prevention (CDC) recommends the use of cryotherapy. Liquid nitrogen, rather than a cryoprobe, should be used to avoid possible vaginal perforation and subsequent fistula formation. An alternative is the use of trichloroacetic acid or bichloroacetic acid carefully applied to the lesions to avoid damage to adjacent tissue. Interferon is no longer recommended for routine use in treating genital warts, due to a high frequency of systemic adverse effects.

Which one of the following is the most reliable clinical symptom of uterine rupture? (check one) A. Sudden, tearing uterine pain B. Vaginal bleeding C. Loss of uterine tone D. Fetal distress

D. Fetal distress. Fetal distress has proven to be the most reliable clinical symptom of uterine rupture. The "classic" signs of uterine rupture such as sudden, tearing uterine pain, vaginal hemorrhage, and loss of uterine tone or cessation of uterine contractions are not reliable and are often absent. Pain and bleeding occur in as few as 10% of cases. Even ruptures monitored with an intrauterine pressure catheter fail to show loss of uterine tone. Signs of fetal distress are often the only manifestation of uterine rupture.

A 16-year-old sexually active nulliparous white female complains of pelvic pain and vaginal discharge. On examination she is found to have a temperature of 39.8 degrees C (102.0 degrees F), pain with movement of the cervix, and tenderness and a mass in the right adnexa. According to CDC guidelines, which one of the following treatments would be appropriate? (check one) A. Outpatient treatment with penicillin G procaine (Wycillin) intramuscularly; probenecid (Benemid) orally; plus doxycycline (Vibramycin) orally for 14 days and reexamination in 3 days B. Outpatient treatment with ceftriaxone (Rocephin) intramuscularly; probenecid orally; plus doxycycline twice a day for 14 days and reexamination in 1 week C. Outpatient treatment with cefoxitin (Mefoxin) intramuscularly; plus doxycycline twice a day for 14 days and reexamination in 10 days D. Hospitalization for treatment with cefoxitin intravenously and doxycycline orally or intravenously, then doxycycline orally twice a day to complete 14 days of treatment

D. Hospitalization for treatment with cefoxitin intravenously and doxycycline orally or intravenously, then doxycycline orally twice a day to complete 14 days of treatment. Patients with PID and tubo-ovarian abscess and high fever should be hospitalized and treated for at least 24 hours with intravenous antibiotics. Amoxicillin and penicillin G procaine are no longer recommended because of the increasing prevalence of penicillinase-producing and chromosomally-mediated resistant Neisseria gonorrhoeae. If cefoxitin is used intramuscularly for outpatient treatment, it should be combined with probenecid. If ceftriaxone is used for outpatient treatment, probenecid is not required. Reexamination should be done within 3 days of initiation of therapy.

A 53-year-old white female visits your office for her annual examination. During the last year she has stopped having menstrual periods and has had moderately severe sleep disturbance. She has been waking up at night with sweats. She denies other problems or complaints. She has a previous history of depression and her family history is significant for osteoporosis, heart disease, and Alzheimer's disease in older members of her family. There is no family history of breast cancer. The patient is concerned about her future and current health and wants to know the benefits and risks of hormone replacement therapy (HRT). Which one of the following statements about HRT is correct? (check one) A. It protects against coronary heart disease B. It slows progression of Alzheimer's disease C. It improves symptoms of depression D. It improves vaginal dryness E. It improves urinary incontinence

D. It improves vaginal dryness. Hormone replacement therapy (HRT) improves the urogenital symptoms of menopause, such as vaginal dryness and dyspareunia. However, recent research regarding HRT has not shown a benefit for reducing coronary events, slowing the progression of Alzheimer's disease, improving depression, or improving urinary incontinence.

A 32-year-old African-American female presents with a 3-day history of fever, cough, and shortness of breath. She has been healthy otherwise, except for a sinus infection 2 months ago treated with amoxicillin. She does not appear toxic. A chest radiograph reveals an infiltrate in the right lower lobe, consistent with pneumonia. Which one of the following would be the best choice for antibiotic treatment? (check one) A. High-dose amoxicillin B. Azithromycin (Zithromax) C. Doxycycline D. Levofloxacin (Levaquin) E. Cefuroxime axetil (Ceftin)

D. Levofloxacin (Levaquin). For previously healthy patients with community-acquired pneumonia and no risk factors for drug resistance, a macrolide such as azithromycin is the preferred treatment (SOR A). Doxycycline is also acceptable (SOR C). Patients who have been treated with antibiotics within the previous 3 months should be treated with a respiratory fluoroquinolone (moxifloxacin, gemifloxacin, or levofloxacin) (SOR A). A β-lactam plus a macrolide is also an alternative (SOR A). The antibiotic chosen should be from a different class than the one used for the previous infection. These alternative treatments are also recommended for those with comorbidities such as chronic heart, lung, liver, or renal disease; diabetes mellitus; alcoholism; malignancies; asplenia; immunosuppressing conditions or use of immunosuppressing drugs; or other risk factors for drug-resistant Streptococcus pneumoniae infection (SOR A).

A 16-year-old female presents with a complaint of pelvic cramps with her menses over the past 2 years. She describes her periods as heavy, and says they occur once a month and last for 7 days, with no spotting in between. She has never been sexually active and does not expect this to change in the foreseeable future. An abdominal examination is normal. Which one of the following would be the most appropriate next step? (check one) A. A pelvic examination B. Ultrasonography C. A TSH level D. Naproxen prior to and during menses

D. Naproxen prior to and during menses. This patient is experiencing primary dysmenorrhea, a common finding in adolescents, with estimates of prevalence ranging from 20% to 90%. Because symptoms started at a rather young age and she has pain only during menses, endometriosis or other significant pelvic pathology is unlikely. An infection is doubtful, considering that she is not sexually active and that symptoms have been present for 2 years. In the absence of red flags, a pelvic examination, laboratory evaluation, and pelvic ultrasonography are not necessary at this time. However, they can be ordered if she does not respond to simple treatment. NSAIDs such as naproxen have a slight effect on platelet function, but because they inhibit prostaglandin synthesis they actually decrease the volume of menstrual flow and lessen the discomfort of pelvic cramping. Acetaminophen would have no effect on prostaglandins.

Which one of the following is an effective screening method for ovarian cancer in elderly females at average risk? (check one) A. Annual CA-125 assays B. Annual pelvic ultrasonography C. Annual Papanicolaou (Pap) tests and pelvic examinations D. No currently available method

D. No currently available method. Two large European trials studied the use of CA-125 and CA-125 with transvaginal ultrasonography (TVU) as screening methods for ovarian cancer. TVU has been reviewed separately. None of these methods is effective as a screening test. No major organization recommends screening women at average risk. The American Cancer Society does not recommend routine screening; the American College of Obstetricians and Gynecologists recommends against population-based screening; an NIH consensus conference recommended obtaining a family history and performing annual pelvic examinations. The U.S. Preventive Services Task Force graded routine screening for ovarian cancer as a "D," meaning that there is fair evidence to recommend excluding ovarian cancer screening as a part of the periodic health examination. This recommendation reflects both a lack of benefit from screening and the fact that a significant number of women have to undergo exploratory surgery to find a single case.

A 24-year-old female with a past history of asthma presents to the emergency department with an asthma exacerbation. Treatment with an inhaled bronchodilator and ipratropium (Atrovent) does not lead to significant improvement, and she is admitted to the hospital for ongoing management. On examination she is afebrile, her respiratory rate is 24/min, her pulse rate is 92 beats/min, and oxygen saturation is 92% on room air. She has diffuse bilateral inspiratory and expiratory wheezes with mild intercostal retractions. Which one of the following should be considered in the acute management of this patient? (check one) A. Chest physical therapy B. Inhaled fluticasone/salmeterol (Advair) C. Oral azithromycin (Zithromax) D. Oral prednisone E. Oral theophylline

D. Oral prednisone. Hospital management of acute exacerbations of asthma should include inhaled short-acting bronchodilators in all patients. Systemic corticosteroids are recommended for all patients admitted to the hospital. The efficacy of oral prednisone has been shown to be equivalent to that of intravenous methylprednisolone (SOR A). Oxygen should also be considered in most patients. Antibiotics are not recommended in the treatment of asthma exacerbations unless there is a comorbid infection. Inhaled ipratropium bromide is recommended for treatment in the emergency department, but not in the hospital (SOR A). Chest physical therapy and methylxanthines are not recommended in the treatment of acute asthma exacerbations.

A male infant is delivered at 41 weeks gestation by spontaneous vaginal delivery. The amniotic fluid is meconium stained. Apgar scores are 7 at 1 minute and 7 at 5 minutes. The baby is noted to have respiratory distress from birth and is hypoxic by pulse oximetry. Respiration improves with supplemental oxygen, as does the hypoxia, but does not return to normal. Which one of the following would most likely be seen on a chest radiograph? (check one) A. A normal heart and lungs B. Fluid in the pulmonary fissures C. Homogeneous opaque infiltrates with air bronchograms D. Patchy atelectasis

D. Patchy atelectasis. The chest radiograph of a child with meconium aspiration syndrome will show patchy atelectasis or consolidation. If the child has a normal chest film and respiratory distress, a noncardiopulmonary source should be considered (i.e., a neurologic or metabolic etiology). The chest film of a child with transient tachypnea of the newborn will show a wet silhouette around the heart, diffuse parenchymal infiltrates, or intralobar fluid accumulation. Homogeneous opaque infiltrates with air bronchograms on a chest radiograph are seen with hyaline membrane disease.

You have provided care for a 27-year-old married, monogamous female for several years. One year ago, she had abnormal cervical cytology that was interpreted as "atypical squamous cells of undetermined significance" (ASC-US). She had repeat cervical cytologic examinations 6 months ago and again last week, both reported as negative. Which one of the following would be the most appropriate next step? (check one) A. Repeat cervical cytology again in 4-6 months B. Screening for human papillomavirus C. Colposcopic examination with a biopsy and endocervical curettage D. Resuming a routine screening protocol E. Cervical culture for herpesvirus

D. Resuming a routine screening protocol. The cervical cytology category of atypical squamous cells of undetermined significance (ASC-US) is one that is poorly reproducible, having been shown to be frequently downgraded to negative or upgraded to a low- or high-grade squamous intraepithelial lesion on review. Recommended management strategies for women with ASC-US include repeat cytology at 4-6 months, immediate colposcopy, and reflex DNA testing for oncogenic HPV types. Should two repeat cytologic examinations at 4- to 6-month intervals prove negative, the patient can safely return to routine cytologic screenings. Should any repeat examination detect ASC-US or more significant cytology, colposcopy is indicated.

A patient at 40 weeks' gestation has had a fundal height 3-4 cm greater than expected relative to dates for the last several visits. Ultrasonography 2 days ago showed a fetus in the vertex position with an estimated fetal weight of 4200 g (9 lb 4oz). On examination today the patient's cervix is closed, long, posterior, and firm, with the vertex at -2 station. Her pregnancy has been otherwise uncomplicated. Appropriate management at this point would be: (check one) A. Cesarean section B. Induction of labor with oxytocin (Pitocin) C. Cervical ripening with prostaglandins D. Scheduling a routine prenatal visit in 1 week

D. Scheduling a routine prenatal visit in 1 week. Fetal macrosomia at term is defined by various authorities as birth weight above 4000-4500 g. Ultrasonography, unfortunately, does not provide a particularly accurate estimate of fetal weight for large fetuses. The risk of difficult vaginal delivery and shoulder dystocia does increase with birth weight above 4000-4500 g. This has led to attempts to prevent shoulder dystocia and possible birth injury by either performing an elective cesarean section or inducing labor when the fetus is estimated to be macrosomic. However, no studies have shown a benefit to either intervention in otherwise uncomplicated pregnancies. Suspected macrosomia on its own is no longer considered an indication for induction or cesarean section. However, should this patient not spontaneously go into labor she will soon need to be managed as a post-dates pregnancy and thus a return visit should be scheduled in a week.

Which one of the following is an indication for a second dose of pneumococcal polysaccharide vaccine (Pneumovax 23) in children? (check one) A. Cerebrospinal fluid leak B. Cyanotic congenital heart disease C. Type 1 diabetes mellitus D. Sickle cell disease E. Chronic bronchopulmonary dysplasia

D. Sickle cell disease. Patients with chronic illness, diabetes mellitus, cerebrospinal fluid leaks, chronic bronchopulmonary dysplasia, cyanotic congenital heart disease, or cochlear implants should receive one dose of pneumococcal polysaccharide vaccine after 2 years of age, and at least 2 months after the last dose of pneumococcal conjugate vaccine (Prevnar 13). Revaccination with polysaccharide vaccine is not recommended for these patients. Individuals with sickle cell disease, those with anatomic or functional asplenia, immunocompromised persons with renal failure or leukemia, and HIV-infected persons should receive polysaccharide vaccine on this schedule and should be revaccinated at least 5 years after the first dose.

A rural community college has requested your guidance in offering a preventive health program to its students. The most appropriate plan would include which one of the following? (check one) A. Mammograms for female students B. Lead poisoning screening for all students C. Stool occult blood kits for students D. Smoking cessation programs E. An annual routine physical examination for all students

D. Smoking cessation programs. The U.S. Preventive Services Task Force recommends a routine physical examination every 3-5 years for young adults until the age of 40. Mammograms are not recommended until age 40. Lead screening is recommended for at-risk individuals between 6 months and 6 years of age. Colorectal cancer screening for average-risk individuals is recommended at age 50. Counseling on tobacco use and other substance abuse is recommended as part of all routine preventive care.

An 83-year-old male has a long history of COPD. His resting oxygen saturation is 86% on room air. Treatment includes oral bronchodilators, inhaled corticosteroids, inhaled beta-agonists, inhaled cholinergics, and home oxygen. Which one of his treatments has been shown to prolong survival in cases such as this? (check one) A. Oral bronchodilators B. Inhaled corticosteroids C. Inhaled beta-agonists D. Inhaled cholinergics E. Home oxygen

E. Home oxygen. Treatment of hypoxemia is critical in the management of COPD and trials have shown a reduction in mortality with the use of oxygen for 15 or more hours daily. Inhaled beta-adrenergic agonists and cholinergic agents, either alone or in combination, provide symptomatic relief but do not prolong survival. Theophylline can be used for symptoms inadequately relieved by bronchodilators. Inhaled corticosteroids do not appear to alter the rate of decline in lung function in COPD. However, some evidence shows that these agents alleviate symptoms and reduce disease exacerbation. Pulmonary rehabilitation improves quality of life and reduces hospitalizations.

A 4-year-old white female is brought to your office by her mother, who reports that the child recently developed a foul-smelling vaginal discharge. After an appropriate history and general examination, you determine that a genital examination is necessary. Which one of the following positions is most likely to allow for visualization of the child's vagina and cervix without instrumentation? (check one) A. Supine in the mother's lap B. The left lateral decubitus position on an examination table C. Trendelenburg's position on an examination table D. The knee-chest position on an examination table E. Supine with the knees spread apart on an examination table

D. The knee-chest position on an examination table. The knee-chest position has been found to allow for visualization of the vagina and cervix of a prepubertal child after 2 years of age without instrumentation. The vagina is filled with air when the child is in the knee-chest position, facilitating inspection. An assistant holds the child's buttocks apart and the child is asked to relax her abdominal muscles and take a few deep breaths. With these preliminary steps, the vaginal orifice opens and the short vaginal canal fills with air. A bright light will help to illuminate the prepubertal child's vagina and cervix. Inspection of genitalia (where examination of the vaginal canal and cervix are not indicated) during a general physical examination need not be in the knee-chest position. In the young child (usually less than 2 years of age), examination is best done with the child lying supine in the mother's lap. For the older prepubertal child, examination is best done with the child lying supine with the knees spread apart on the examination table. The other positions listed are not helpful or recommended when examining the genital area of a prepubertal child.

The most common cause of abnormal vaginal discharge in a sexually active 19-year-old female is (check one) A. Candida albicans B. Trichomonas vaginalis C. Staphylococcus D. Group B Streptococcus E. Bacterial vaginosis

E. Bacterial vaginosis. Bacterial vaginosis (BV) is the most common cause of acute vaginitis, accounting for up to 50% of cases in some populations. It is usually caused by a shift in normal vaginal flora. BV is considerably more common as a cause of vaginal discharge than C. albicans and T. vaginalis.

The treatment of choice for a 4-month-old infant with suspected pertussis is: (check one) A. Supportive care (respiratory, fluids) only B. Ceftriaxone (Rocephin) C. Ampicillin D. Gentamicin (Garamycin) E. Erythromycin

E. Erythromycin. In spite of widespread vaccination of infants, pertussis occurs endemically in 3- to 5-year cycles in the U.S. It appears to be more common within populations not routinely immunized, such as Mennonite communities, but can occur widely. Infants younger than 6 months are affected most severely, although pertussis occurs in all age groups. The diagnosis is made by nasopharyngeal culture, but because the disease is uncommon and the organism is fastidious, laboratory personnel should be advised of the physician's suspicion of pertussis. Treatment includes respiratory and nutritional supportive care, particularly for infants younger than 6 months. Antibiotic therapy is most effective in shortening the illness when given early, during the upper respiratory phase, but is indicated at any stage to reduce the spread of disease to others. The drug of choice is erythromycin, 40-50 mg/kg/day divided into four doses, for 14 days. Also effective are azithromycin and clarithromycin, which may be better tolerated with improved compliance. Resistance to these agents is rare. Penicillins and cephalosporins are ineffective. Gentamicin is potentially very toxic, and is not indicated.

A 17-year-old white female visits you for a physical examination prior to entering college. During the review of systems her only complaint is cyclic lower abdominal cramps around the onset of menstruation. She reports that pain has been present to some degree with most of her periods since about 6 months after menarche. The pain is often severe enough for her to miss school. Each episode lasts 24-48 hours and is somewhat relieved by rest and acetaminophen. Her menstrual history is otherwise normal. She denies ever being sexually active and tells you that she has received little empathy from her mother, who had similar symptoms as an adolescent that improved after her first pregnancy. Pelvic and rectal examinations are within normal limits. Which one of the following management choices would be appropriate at this time? (check one) A. Referral for hysterosalpingography B. Referral for psychological counseling C. Danazol (Danocrine) D. Acetaminophen/hydrocodone (Vicodin HP) E. Naproxen sodium (Anaprox)

E. Naproxen sodium (Anaprox). The patient's history is typical of primary dysmenorrhea, defined as severe cramping pain in the lower abdomen that occurs during menses; it may also occur prior to the onset of menses in the absence of associated pelvic pathology. Although many women complain of pain beginning with the first cycle, symptoms usually begin at the onset of ovulation around 6-12 months after menarche. Symptoms typically last 48 hours or less, but sometimes may last up to 72 hours. It is common to find daughters with dysmenorrhea whose mothers had the same symptoms. Additionally, the symptoms of primary dysmenorrhea often resolve after the first pregnancy. In this patient, who has no history suggesting an emotional disorder, there is no need for psychological counseling at this time. Further evaluation could include ultrasonography to rule out causes of dysmenorrhea such as uterine leiomyomata, adnexal masses, and endometrial polyps. However, a trial of symptomatic therapy is most reasonable before other invasive studies, such as a laparoscopic examination or a hysterosalpingogram, are ordered. It is not reasonable to begin danazol without a diagnosis of endometriosis, which is by definition secondary dysmenorrhea. Since neither inhibits prostaglandin synthetase, acetaminophen (which she had already tried without complete relief) combined with a narcotic is not an appropriate management strategy. Multiple placebo-controlled studies have shown that NSAIDs such as naproxen, at the onset of symptoms, provide significant relief of primary dysmenorrhea compared to placebo.

The Centers for Disease Control and Prevention (CDC) recommends antenatal screening for group B streptococcal disease by: (check one) A. Culturing the urine at 20 weeks' gestation B. Obtaining cultures from the rectum and vaginal introitus at 20 weeks' gestation C. Obtaining a culture from the cervix at 35-37 weeks' gestation D. Obtaining cultures from the cervix and rectum at 35-37 weeks' gestation E. Obtaining cultures from the rectum and vaginal introitus at 35-37 weeks' gestation

E. Obtaining cultures from the rectum and vaginal introitus at 35-37 weeks' gestation. The gastrointestinal tract is the most likely reservoir of group B Streptococcus with secondary spread to the genital tract. Cultures from the vaginal introitus and the rectum are the most sensitive for detecting colonization. No speculum examination is necessary. The closest time to delivery that cultures can be performed and allow time for results to be available is 35-37 weeks' gestation. Culture-positive women are then treated during labor. Other criteria for the use of chemoprophylaxis during delivery continue to apply.

A 12-year-old Hispanic female is brought to your office because of the recent onset of a white vaginal discharge. She is otherwise asymptomatic and has never menstruated. She denies sexual activity and a general examination reveals no abnormalities. You note the presence of breast buds and scant pubic hair. Microscopic examination of the vaginal discharge shows sheets of vaginal epithelial cells. Which one of the following is most likely in this setting? (check one) A. Pinworm (Enterobius vermicularis) infestation B. Sexual abuse C. Vaginal foreign body D. Trichomoniasis E. Physiologic secretions

E. Physiologic secretions. This child is entering puberty. In the 6- to 12-month period before menarche, girls often develop a physiologic vaginal discharge secondary to the increase in circulating estrogens. The gray-white discharge is non-irritating. When physiologic discharge is examined with the microscope, sheets of vaginal epithelial cells are seen. The only treatment necessary is reassurance of both parents and child that this is a normal process that will subside with time. The other conditions listed are pathologic and have other associated symptoms and findings not seen in this case. Pinworms normally cause perianal and vulvar pruritus and irritation. The findings in sexual abuse range from an inflamed vulvovaginal area, to evidence of sexually transmitted diseases, to evidence of local trauma. Trichomoniasis would cause vulvovaginal irritation and microscopic examination of the discharge would show Trichomonas organisms. A vaginal foreign body would usually present with a foul and/or bloody vaginal discharge.

A 32-year-old gravida 3 para 2 is in labor at term following an uncomplicated prenatal course. As you deliver the fetal head it retracts against the perineum. Downward traction fails to free the anterior shoulder. The most appropriate course of action would be to: (check one) A. Apply increasingly strong downward traction to the fetal head B. Have an assistant apply fundal pressure C. Deliberately fracture the clavicle of the fetus D. Begin an intravenous nitroglycerin drip E. Place the mother's thighs on her abdomen

E. Place the mother's thighs on her abdomen. While there are several risk factors for shoulder dystocia, most cases occur in fetuses of normal birth weight and are not anticipated. Once it does occur, excessive force should not be applied to the fetal head or neck and fundal pressure should be avoided, as these manuevers are unlikely to free the fetus and can injure both mother and infant. Up to 40% of shoulder dystocia cases can be successfully treated with the McRoberts maneuver, in which the maternal hips are flexed and abducted, placing the thighs up on the abdomen. Adding suprapubic pressure can resolve about half of all shoulder dystocias. Additional maneuvers include internal rotation, removal of the posterior arm, and rolling the patient over into the all-fours position. As a last resort, one can deliberately fracture the fetal clavicle, perform a cesarean section with the vertex being pushed back into the birth canal, or have the surgeon rotate the infant transabdominally with vaginal extraction performed by another physician. General anesthesia or nitroglycerin, orally or intravenously, may be used to achieve musculoskeletal or uterine relaxation. Intentional division of the cartilage of the symphysis under local anesthesia has been used in developing countries, but should be used only if all other maneuvers have failed and a cesarean delivery is not feasible.

A 47-year-old gravida 3 para 3 is seen for a physical examination. She has had a total abdominal hysterectomy for benign uterine fibroids. Which one of the following is the recommended interval for Papanicolaou (Pap) screening in this patient? (check one) A. Every 5 years B. Every 3 years C. Every 2 years D. Annually E. Routine screening is not necessary

E. Routine screening is not necessary. Most American women who have undergone hysterectomy are not at risk of cervical cancer, as they underwent the procedure for benign disease and no longer have a cervix. U.S. Preventive Services Task Force recommendations issued in 1996 stated that routine Papanicolaou (Pap) screening is unnecessary for these women. Nevertheless, data from the Behavioral Risk Factor Surveillance System (1992-2002) indicated that in the previous 3 years, some 69% of women with a previous history of hysterectomy for benign causes had undergone screening.

Late decelerations on fetal monitoring are thought to indicate which one of the following? (check one) A. Fetal head compression B. Umbilical cord compression C. Fetal sleep D. Uterine hypotonus E. Uteroplacental insufficiency

E. Uteroplacental insufficiency. Late decelerations are thought to be associated with uteroplacental insufficiency and fetal hypoxia due to decreased blood flow in the placenta. This pattern is a warning sign and is associated with increasing fetal compromise, worsening fetal acidosis, fetal central nervous system depression, and/or direct myocardial hypoxia. Early decelerations are thought to result from vagus nerve response to fetal head compression, and are not associated with increased fetal mortality or morbidity. Variable decelerations are thought to be due to acute, intermittent compression of the umbilical cord between fetal parts and the contracting uterus.

An 18-year-old single white female at 30 weeks' gestation presents to the hospital with uterine contractions 10 minutes apart. Her previous pregnancy 18 months ago resulted in a preterm birth at 29 weeks' gestation. The most accurate test to determine whether this patient will need hospitalization and tocolysis would be: (check one) A. Serum corticotropin-releasing hormone B. Maternal serum alpha-fetoprotein C. Serum human chorionic gonadotropin (hCG) D. Salivary estriol concentration E. Vaginal fetal fibronectin

E. Vaginal fetal fibronectin. Of the biochemical markers listed, the most clinically useful test to differentiate women who are at high risk for impending preterm delivery from those who are not is the fetal fibronectin in cervical or vaginal secretions. In symptomatic women, this is most accurate in predicting spontaneous preterm delivery within 7-10 days. It is less accurate in those who are asymptomatic. If the fetal fibronectin is negative, it may be possible to avoid interventions such as hospitalization, tocolysis, and corticosteroid administration.


Set pelajaran terkait

Number system, binary, decimal, hexadecimal, octal

View Set

Introduction to Micro Economics: Final Exam

View Set

פסחים פרק ב משנה ו

View Set

Chapter 13: Intravenous Therapuy

View Set

Chapter 2 The Internet, The Web, and Electronic Commerce

View Set